Last visit was: 31 Aug 2024, 17:26 It is currently 31 Aug 2024, 17:26
Close
GMAT Club Daily Prep
Thank you for using the timer - this advanced tool can estimate your performance and suggest more practice questions. We have subscribed you to Daily Prep Questions via email.

Customized
for You

we will pick new questions that match your level based on your Timer History

Track
Your Progress

every week, we’ll send you an estimated GMAT score based on your performance

Practice
Pays

we will pick new questions that match your level based on your Timer History
Not interested in getting valuable practice questions and articles delivered to your email? No problem, unsubscribe here.
Close
Request Expert Reply
Confirm Cancel
SORT BY:
Date
   1  ...  7   8   9   10   11  ...  21   
Manhattan GMAT Online Marketing Associate
Joined: 14 Nov 2013
Posts: 272
Own Kudos [?]: 51 [0]
Given Kudos: 0
Send PM
Manhattan GMAT Online Marketing Associate
Joined: 14 Nov 2013
Posts: 272
Own Kudos [?]: 51 [0]
Given Kudos: 0
Send PM
Manhattan GMAT Online Marketing Associate
Joined: 14 Nov 2013
Posts: 272
Own Kudos [?]: 51 [0]
Given Kudos: 0
Send PM
Manhattan GMAT Online Marketing Associate
Joined: 14 Nov 2013
Posts: 272
Own Kudos [?]: 51 [0]
Given Kudos: 0
Send PM
Here’s why you should take the GMAT twice. [#permalink]
Expert Reply
FROM Manhattan GMAT Blog: Here’s why you should take the GMAT twice.


Over the past five or so years, I have seen more and more students take the GMAT twice.

Now that students can cancel a score and never have it appear on record, I’ve come to the recommendation that everyone should plan to take the GMAT twice.

Taking the GMAT is seriously stressful.
Most people become at least a little nervous when taking any standardized test. A computer adaptive test is even more stressful because, no matter how much you study, the test just keeps getting harder as you learn more.

I’ve had many students take the test twice, and every single one has told me that they felt more comfortable the second time. They knew what to expect at the testing center, the security procedures didn’t stress them out, and they were even better able to handle the small distractions of the testing room—another student typing, a proctor entering the room, and so on.

For those who know that they get extra nervous when taking standardized tests, having a “dry run” first test is a great way to help keep a handle on your nerves when you take the test “for real,” the second time.

There’s no downside: MBA programs use your highest score. 
Most people have heard that business schools use your highest score, but many people don’t trust that the schools really don’t care about lower scores on your record. In fact, some schools don’t even look at your full set of scores until they’ve decided what to do with your application (and this used to be true for all schools). Let me take you through that process. (Note: this applies to MBA programs. If you are going for a Ph.D., the process may be different and the doctoral program may care about all of your scores.)

Let’s use an example to illustrate what’s going on. Last year, Stanford’s Graduate School of Business* received approximately 8,000 applications spread over 3 rounds of admission. The school admitted just over 400 people, or about 5% of applicants (yikes!).

Now, picture the offices of the admissions people. They have mounds of information to get through: essays, recommendations, resumes. They know they’ll admit only about 5% of the people who’ve applied. Do you think they’re going to check the official GMAT score reports of all 8,000 people?

No way! Instead, they evaluate the applications assuming that you told them the truth when you reported a certain GMAT score. Once they’ve generally decided who they want to admit (or put on the wait list), then they’ll verify the scores just for those students.

In the past few years, some schools have built in the ability to link your account to your test results after you enter certain pieces of identifying info into your application. If so, then you’ll see your verified test date(s) pop up in your official application at this point. (*Note: I don’t know how Stanford does things; I chose them for the example above because they have an exceptionally low yield, so they make for an especially good example.)

If they don’t have access to your full score set at the beginning, then they’ve already decided your fate by the time they look up your scores. If they want you, they’re not suddenly going to reject you because you had another GMAT score that was lower. After all, you did earn that higher GMAT score on which they based their decision! (Assuming you did. This should be obvious but here goes: don’t lie about your GMAT score on your application.) And even if they do have access to your full score set at the beginning, they’re used to making this decision based on your highest score. That’s how they’ve always done it. If you can get that top score once, then you are capable of getting that score period, and that’s what they care about.

There’s no downside, part 2: you can cancel your scores 
Any canceled test administrations don’t show up on your record at all. The schools literally won’t even know that you took the test that day.

So if you go in and really dislike your score, just select the button to cancel at the end. Then you don’t need to worry about whether some b-school might penalize you for a lower score even if you later earn a higher score. (Though, again, you really don’t need to worry about this!)

Know before you go in what kind of score you’d want to keep vs. cancel. In the 4 months since this new cancellation policy started, I’ve talked to two students who’ve canceled when the screen flashed a 690 and they wanted a 700+. Seriously! They were only 10 points off and they canceled their scores!! Don’t fall prey to a knee-jerk reaction just because the score you wanted isn’t on the screen. (You can reinstate your scores within 2 months of a cancellation…if you pay a $100 fee.)

I generally tell my students that if they score more than 100 points below their (reasonable) goal, then they should feel free to click the cancel button if it makes them feel more comfortable.

Note my “reasonable” caveat. If you want a 730, and your practice tests topped out at 580, and then you score a 620 on the real test, please do not cancel that score. You just had your best test ever and you want to keep that score, just in case.

But the GMAT costs $250…that’s a downside!
I agree that $250 is a lot of money. If you think about how much money you spent the last time you went to dinner, then $250 is definitely expensive.

But put this figure into perspective. If you’re going for a full-time program at a private school, you’re looking at a $200,000 price tag! If you attend a public school via a part-time, evening-and-week-end program, business school is still going to cost you tens of thousands of dollars. The cost of one GMAT, $250, is far less than 1% of the cost of b-school. Don’t try to save $250 now when a proper investment might get you into a higher-caliber program down the line.

So build two tests into your study timeline. If you end up loving your first score, then you can take that second $250 and go out for a really, really nice dinner. 

[Edited] Note: this article was edited after first publication. Initially, I said that many / most schools don’t check scores until after making the decision. It then came to my attention that more schools had added the functionality described in the article than I had thought, so I edited the article to say that some schools don’t check until after and some schools attach the data to the application from the start.


Stacey Koprince is a Manhattan Prep instructor based in Montreal, Canada and Los Angeles, California.
Stacey has been teaching the GMAT, GRE, and LSAT  for more than 15 years and is one of the most well-known instructors in the industry. Stacey loves to teach and is absolutely fascinated by standardized tests. Check out Stacey’s upcoming GMAT courses here



 

The post Here’s why you should take the GMAT twice. appeared first on GMAT.
This Blog post was imported into the forum automatically. We hope you found it helpful. Please use the Kudos button if you did, or please PM/DM me if you found it disruptive and I will take care of it. -BB
Manhattan GMAT Online Marketing Associate
Joined: 14 Nov 2013
Posts: 272
Own Kudos [?]: 51 [0]
Given Kudos: 0
Send PM
Here’s why you might be missing GMAT Data Sufficiency problems (Part 2 [#permalink]
Expert Reply
FROM Manhattan GMAT Blog: Here’s why you might be missing GMAT Data Sufficiency problems (Part 2)


In our previous article, we divided the logical errors that test-takers make on Data Sufficiency questions into two types:

Type 1: You thought that something was sufficient, but it was actually insufficient.

Type 2: You thought that something was insufficient, but it was actually sufficient.

We already covered the most common reasons for Type 1 errors to occur and a few good ways to avoid them; now, let’s cover Type 2 errors.

Type 2 errors are their opposite: also known as “false negatives,” they happen when it looks like you don’t have enough information to answer the DS question, but you actually do. The classic “(C) trap” is one example of a Type 2 error: say that you picked (C), when the right answer was actually (B). In that case, you thought that each statement was insufficient, and decided to combine them. But, it turned out that (2) was actually sufficient on its own. Oops.

According to data from our GMAT Navigator program (in which thousands of students have recorded their answers to retired GMAT DS problems), Type 2 errors happen about 50 percent more often than

Type 1 errors. That’s because DS problems often deliberately include statements that look useless and irrelevant, but really aren’t. If you’re not prepared, when you see one of these problems, you’ll wrongly assume that the statement is insufficient. Only well-prepared test-takers will see through the ruse.

Here are some situations that often cause Type 2 errors:
You mistook a yes/no question for a value question. Did you see a yes/no DS question, but treat it as if you needed to solve for an exact value? Then you likely made a Type 2 error: you incorrectly assumed that because you couldn’t come up with a single value, you couldn’t answer the question. Remember that on yes/no questions, the only answers you’re looking for are ‘yes’ and ‘no’. You don’t need to come up with a specific number, and in fact, problems are often designed so that you can’t! Some yes/no questions that often lead to Type 2 errors are The Official Guide to the GMAT, 2016 (OG 2016) DS 9 and 108, and The Official Guide for GMAT Quantitative Review, 2016 (QR 2016) DS 122.

Not enough math. One rule of thumb says that in order to solve a math problem, you need at least as many equations as you have unknowns. If you’re pressed for time, use that rule on DS word problems and algebra problems. But also know that in certain special circumstances, you can solve by using fewer equations than you’d normally need. The only way to discover these special cases is to translate both the question and the statements into math on your scratch paper, and then decide whether you can solve. Otherwise, you’ll make a Type 2 error–you’ll assume that you can’t solve, when you actually can. Try these DS problems for practice: OG 2016 DS 106 and 140, and QR 2016 DS

Combo traps. Some DS value questions ask you to solve for a combination of values rather than for a single value. For instance, a DS question might ask you “What is the value of xy?” or “What percent of John’s food budget did he spend in restaurants?” In the real world, you’d solve for each individual value first (x and y, or John’s total budget and his restaurant spending). Unfortunately, the test writers design combo questions that you can actually answer without knowing the individual values, and if you fail to notice these questions, you’ll be vulnerable to Type 2 errors. That is, you might not need as much information to solve for a combination of values as you would need to solve for the values separately.

When you review a DS value question, decide whether you were asked for one value or for a combination. If you had to find a combination of values, you may have mistakenly assumed that you needed to solve for both values separately, making a Type 2 error. To avoid this, conclusively prove each statement insufficient before you ever put them together. For some practice, try OG 2016 DS 52, 63, and 97.

“Nice but not necessary.” When you do a DS problem, you’re looking for the minimum amount of information that would let you answer the question correctly. That requires a different sort of thinking than solving real-world problems does.Suppose that a friend asks you to figure out the amount of water in her backyard swimming pool. She knows three things: the current depth of water in the swimming pool, how long it took her to fill it with a hose, and the rate at which her hose puts out water. You’d probably respond by asking her for all of the information she has first, and then you’d start writing equations. You might notice halfway through that you didn’t actually need to know how deep the pool was, but who cares? You’d find the answer, and your friend would walk away happy.On Data Sufficiency, you can’t ask for all of the information and then see what happens. Just because you can definitely answer the question when you use both statements together, doesn’t mean that the statements are insufficient on their own! It might be a little more complicated to answer the question using only the information from one statement, but it could still be possible.

If you made a Type 2 error and then found yourself saying “I didn’t realize that I didn’t have to know that!” about one of the statements, then you fell for a Nice But Not Necessary trap. These questions often have a particular look to them: one statement will be very simple, usually giving you a single value, like the depth of a swimming pool or the time at which a machine began working. The other statement will be much more complex and harder to handle. Be skeptical when you see this–it might be nice to know the information from the simpler statement, but do you really need it? Or is there a more complicated solution that only uses the info from more complex statement? To practice, check out problems OG 2016 DS 38, 89, and 95, and QR 2016 DS 74 and 88.

What now?
With the information from this series of articles , you can categorize your Data Sufficiency errors into four types: Type 1, Type 2, Careless (simple computation mistakes and miswritings), and Mathematical (you didn’t know a rule or applied it incorrectly). Go through some DS problems you’ve done in the past and assign each wrong answer to one of those four categories. What patterns do you notice? Are your mistakes predictable?

Then, focus specifically on your Type 2 errors. Whenever you notice one of these errors, identify the trick that you fell for. The situations described in this article often lead to Type 2 errors, but they aren’t the only possible causes. Can you come up with others? Based on the patterns you notice, make one change to how you do DS problems. Some options:

  • As soon as you read a DS question, write either “value” or “yes/no” on your paper.
  • Always translate DS word problems into variables and equations before deciding that a statement is insufficient.
  • Before you begin solving a DS value problem, determine whether it’s a combo problem or not.
Now that you know what these two types of logical errors look like, start trying to recognize them as you review DS problems. With time and practice, you’ll start to notice problems that would normally lead you to make Type 1 or Type 2 errors as you see them. Understanding why you made a mistake is the first step to avoiding it next time! 


Chelsey Cooley is a Manhattan Prep instructor based in Seattle, Washington.
 Chelsey always followed her heart when it came to her education. Luckily, her heart led her straight to the perfect background for GMAT and GRE teaching: she has undergraduate degrees in mathematics and history, a master’s degree in linguistics, a 790 on the GMAT, and a perfect 170/170 on the GRE. Check out Chelsey’s upcoming GMAT courses here.

The post Here’s why you might be missing GMAT Data Sufficiency problems (Part 2) appeared first on GMAT.
This Blog post was imported into the forum automatically. We hope you found it helpful. Please use the Kudos button if you did, or please PM/DM me if you found it disruptive and I will take care of it. -BB
Manhattan GMAT Online Marketing Associate
Joined: 14 Nov 2013
Posts: 272
Own Kudos [?]: 51 [0]
Given Kudos: 0
Send PM
Make the Most of your Holiday Study Time (Part 1) [#permalink]
Expert Reply
FROM Manhattan GMAT Blog: Make the Most of your Holiday Study Time (Part 1)

The holidays are fast approaching and all of my students are asking me how best to use their extra downtime over the next few weeks. I have some different recommendations, depending upon your circumstances.

In this installment, we’ll talk about what to do if you’re aiming for the second round (in other words, your applications are due in January 2016—soon!). In the next installment, we’ll talk about what to do if this circumstance does not apply to you.

I’m aiming for Round 2 admissions (early January 2016)
If you’re aiming for round 2, then this is obviously going to be a busy holiday season.

Let’s see if we can’t find a way to lessen the stress a bit while still allowing you to accomplish what you need.

You’ve got two broad tasks at hand: taking the GMAT and finishing your applications. You’re going to have to balance your time well so that one of these aspects doesn’t suffer unduly.

Task #1, then, is to take a good hard look at your goal GMAT score. Sure, it would be unbelievably fantastic if you could score a 720, but you have to ask yourself whether that’s a realistic goal in the few weeks that you have left, particularly given that you still have essays and other aspects of your applications to finalize. So what do you think is a realistic goal, given everything you have to do?

Cut your stressful commitments
Next, pull out your calendar. You’re going to have to work on some aspect of this stuff every single day. (As if the holidays weren’t stressful enough. Sorry.) Can you reduce any stressful commitments? Are there holiday parties or family events you really don’t want to attend? Don’t—you have the perfect excuse. Send your regrets right now. Can you “work” from home a couple of days or take time off? Use up some of your sick days? Schedule it. ☺️

Do not, though, skip events that you’re looking forward to; you will need some breaks in the coming weeks. If you burn yourself out, you won’t do very well on the test, so remind yourself that I ordered you to go to your best friend’s awesome annual party and have a great time.

Is the craziness of everything really getting to you? Try meditating to help manage the stress of holidays + GMAT + applications.

Schedule study appointments with yourself
Okay, now start blocking off chunks of time every day from now until your applications are due: 30 minutes here, an hour there, 2 hours over there. Give yourself good breaks between sessions; your brain will thank you. You’re probably going to need a total of 15 to 30 hours a week, depending on how many applications you’re doing and how much you want to lift your GMAT score.

Plan to do some GMAT study and some work on applications every day. This is better than doing all-GMAT today and all-essays tomorrow. Your brain will stay fresher if you mix up your activities.

Plan to take 1 or 2 practice tests. Block off 4 hours and start at the same time of day that you will start the real test. Take the tests at least one week apart. Don’t take a practice test within 5 days of the real test. Make sure you take your tests under 100% official conditions, including the essay and IR sections. (I know you don’t care as much about those scores. But you do care that your practice mimics the real test.)

Fix your timing
The single biggest thing that people consistently mess up on the GMAT is timing. I would bet that almost anyone can pick up 20 to 30 points on the GMAT simply by getting better at knowing what NOT to do. I’m 100% serious!

Knowing what NOT to do is related to business decision-making: both while studying and during the test, where do you want to spend your scarce time (and mental energy) and where don’t you? You don’t invest in every last idea that your employees brainstorm; you pick out the most promising ones and allocate your scarce resources accordingly. By the same token, on the GMAT, you can’t do it all; if you try, you won’t maximize your score on the test.

Got that mindset? Here’s how to put your best foot forward with respect to timing on the test.

Focus on the lowest-hanging fruit
You can’t study it all, either. You have limited time and a split focus (applications!).

You need to prioritize: work on the easiest-to-improve stuff first and just get as far as you can before you run out of time and have to take the test.

Analyze your most recent CAT or your performance on recent practice problems.

Where were you almost there? Use that data to drive your study and review.

The lowest of the low-hanging fruit is careless mistakes. You already know how to do this stuff! You just need to figure out how to minimize whatever types of careless mistakes you’re tending to make. This might be, “Hmm, I solved for the wrong thing on a couple of problems. What am I going to do on every problem from now on to make sure that I solve for the right thing?” Or it might be, “Oh, I used to know how to do that, but I haven’t looked at it in 6 weeks and I’m starting to forget. I need to review that.”

Don’t just tell yourself, “Ugh, don’t do that!” That’s not enough. Figure out why you made that careless mistake and what new habit you can implement to reduce the chances of a repeat of that specific kind of mistake.

Another low-hanging area has to do with holes in foundational-level material. Did you mess up that weird problem with negative exponents and fractions and you’re still not 100% sure what the thing was talking about? Whatever. That’s too hard anyway; blow it off.

But did you mess up that problem that required you to solve a pretty straightforward linear equation? You know, the one where you read the explanation after and thought, I didn’t know that before, but I get it. That’s not that hard. Okay: practice so you can do it next time!

Also take a look at problems that you know how to do, but that tend to take you about 30 to 60 seconds longer than the average expected for that question type. Are there any shortcuts you can learn to shave 15 or 30 seconds? If so, practice implementing them so that you can save up some time to spend elsewhere or just take the timing pressure off a bit.

Find someone to commiserate and motivate
Know anyone else who’s studying right now or working on applications? If you’re both studying, get together two or three times a week to study (even if you’re both mostly working on your own stuff). If you aren’t in the same city, meet via Google Hangout or similar. Tell your friend what you did since the last time you met and what you plan to do until the next time you meet. That’ll help keep you on track, since you know you’ll have to fess up if you don’t do anything!

Even if your friend is already done with the test, you can still ask him or her to be your “keep me on track” study buddy: you say what you’ve done and what you’re going to do each week. He or she went through the GMAT too and will understand.

Get together for happy hour or brunch once a week to decompress and commiserate (again, even if that’s both of you bringing a drink to your couch while you’re on video chat or a phone call).

Do your best in the time you have left.
Give it your best shot. Time is limited and it’s possible that you might not make everything come together by round 2—but at least you’ll know that you’ve really tried. You can always decide to postpone your application and try again. Good luck and happy studying!

Join us next time, when we’ll talk about how to make the most of your holiday season if you’re not planning to apply during round 2 this year.


Stacey Koprince is a Manhattan Prep instructor based in Montreal, Canada and Los Angeles, California.
Stacey has been teaching the GMAT, GRE, and LSAT  for more than 15 years and is one of the most well-known instructors in the industry. Stacey loves to teach and is absolutely fascinated by standardized tests. Check out Stacey’s upcoming GMAT courses here.



 

The post Make the Most of your Holiday Study Time (Part 1) appeared first on GMAT.
This Blog post was imported into the forum automatically. We hope you found it helpful. Please use the Kudos button if you did, or please PM/DM me if you found it disruptive and I will take care of it. -BB
Manhattan GMAT Online Marketing Associate
Joined: 14 Nov 2013
Posts: 272
Own Kudos [?]: 51 [0]
Given Kudos: 0
Send PM
Here’s how to use the holidays for GMAT prep (Part 2) [#permalink]
Expert Reply
FROM Manhattan GMAT Blog: Here’s how to use the holidays for GMAT prep (Part 2)

Last time, we talked about how to use holiday downtime to get ready for round 2 GMAT admissions. This time, we’re going to talk about what to do if you aren’t applying for round 2 this year.

We have two broad scenarios to talk about:

1. Do you work an 80-hour-a-week job or have some other significant commitments during non-holiday time such that your studying is suffering due to lack of time?

2. Do you have some downtime around the holidays, or at least less-crazy time?

If you answered “yes” to both of the above questions, then you’re going to use the holidays to give your studies a much-needed boost.

If, on the other hand, you’ve been studying at least 10 hours a week (and really more like 15), and you’re generally keeping your studies on track during non-holiday time, then here’s your reward: you’re going to slow down during the holidays and give your brain a much-needed break, so that you will be fresher when you ramp up again after the holidays.

Scenario #1: My normal schedule is crazy but I have some breathing room around the holidays.
The bad news is that you haven’t been getting as much done as you need, but the good news is that you can use your downtime over the holidays to get more done. The driving concept: prioritization.

You do not want to turn your 80-hour regular week into an 80-hour study week. Your brain can learn only so many new things at once; if you overload it, it will start to rebel and “drop” memories. (You’ve experienced this, right? You can’t remember what was decided at that meeting 3 days ago. Your significant other swears s/he asked you to pick up milk on the way home, but you have no memory of that exchange…until you find the evidence on your phone. ☺️)

You’re going to follow some of the advice that I discussed in the first installment of this article (so pull that up right now), with a few differences. Pull out your calendar and follow the advice I gave with respect to scheduling study appointments with yourself. Since you don’t have a rapidly-approaching deadline to take the real test, though, plan to take only one practice test.

If it has been more than about 4-6 weeks since your last practice CAT, plan to start your holiday study period with a CAT. If you have taken a CAT fairly recently, plan to end your holiday study period with a CAT.

Either way, analyze your most recent test and classify everything into your three buckets (you’ll understand what buckets are after you have read that article). You’re going to concentrate on bucket #2.

Important note: bucket #3 is for things that are so hard for you that you’re going to blow them off right now—literally, just get them wrong fast. People hesitate to put things in bucket #3 because they don’t want to give up on that category forever. That’s not what you’re doing here! You’re just saying, “Right now, given my limited time, this isn’t worth my time in the next few weeks. I have other, better lower-hanging-fruit that I’m going to do first.” Later, you’ll revisit your bucket 3 and move some things to bucket 2 as bucket 2 items move to bucket 1.

Next, you’re going to work intensively on your bucket 2 items over the holidays.

Use the process described here to minimize careless mistakes.

For holes in your foundation (easier problems that you’re missing), start with the relevant chapters in our Foundations of Math (FoM) or Foundations of Verbal (FoV) strategy guides (or equivalent study materials from other companies). First plug any holes in your foundational knowledge. When you’re ready, move up to the equivalent chapters in the main strategy guides (or, again, equivalent materials from others). Finally, test your new knowledge and understanding by trying problems from the Official Guide.

For example, if you’re struggling with exponents and roots, start with those chapters in FoM. Make flash cards for anything you need to memorize. Use the end-of-chapter and online problem sets to drill your skills. Then move into the Algebra strategy guide and dig into the more advanced exponent and root material there. Again, make flash cards and use the end-of-chapter and online problem sets to drill. Finally, try a few (2-3) OG problems from these areas, and make sure that you’re analyzing these problems at the 2nd Level.

This is important: do NOT do all of the OG exponent problems at once. Half of the battle on the GMAT is figuring out what a new problem is testing; on the real test, you’ll never know that you’re about to get an exponent problem. So most of your OG study needs to be done in timed sets of mixed questions, where you’re having to jump around, figure out for yourself what each new problem is, and decide how to allocate your time and mental energy among the questions.

After 2-3 weeks, work will ramp back up again. Take another CAT to gain experience and gauge your progress. Analyze using that same article linked earlier and revisit your buckets to see what you can move from bucket 2 to bucket 1 and whether you want to move anything from bucket 3 to bucket 2. (Note: some things really should stay in bucket 3 forever. For me, combinatorics and 3-D geometry will always be bucket 3.)

Scenario #2: My regular study time is already pretty productive.
That’s great! Now here’s your reward. Yes, you’re going to study some over the holidays, but you’re actually going to give your brain a pretty substantial break. I wish that I could recommend this to everyone, but the realities for some (crazy jobs, looming deadlines) make this impossible.

Pull up your calendar. Block out around 5 days completely; you aren’t even going to think about the GMAT on these days. Seriously! You’ve been working and studying hard for a while now, and your brain is becoming fatigued. This break is going to allow you to come back super fresh and re-energized in January.

If you feel guilty when those days come along, say to yourself, “Stacey said not to think about the GMAT today!” and go merrily about your day. ☺️

What you are going to do on some days during this break is organize. Do you have notes all over the place about SC rules? Consolidate them into one file or notebook.

Are you not quite sure which lessons you’ve done thoroughly and which weren’t done as well the first time around? Are you not sure what you need to review? Take some time to look over previous lessons just with an eye towards classifying them: this one is good; this other one could use some light review; I need to re-do this lesson from start to finish. You don’t actually have to do them now; you’re just figuring out what you’re going to do when you ramp back up again after the holidays.

Here’s the one substantial thing you can do: if it’s been more than a month since you’ve last taken a practice CAT, take one over the holidays and analyze it. Also analyze the individual questions (you might analyze just the ones you got wrong now and save the other ones for later). Altogether, this assignment might take you around 8 to 15 hours, depending on whether you analyze everything now.

Use this analysis to help you figure out what you need to prioritize after the holidays; don’t actually start studying now.

If you find yourself falling between two of these categories, mix and match the advice from each one. Alternatively, come talk to me on the MPrep forums and ask for advice regarding your particular situation.

Good luck, happy end-of-2015, and see you in the New Year!


Stacey Koprince is a Manhattan Prep instructor based in Montreal, Canada and Los Angeles, California.
Stacey has been teaching the GMAT, GRE, and LSAT  for more than 15 years and is one of the most well-known instructors in the industry. Stacey loves to teach and is absolutely fascinated by standardized tests. Check out Stacey’s upcoming GMAT courses here.



 

 

The post Here’s how to use the holidays for GMAT prep (Part 2) appeared first on GMAT.
This Blog post was imported into the forum automatically. We hope you found it helpful. Please use the Kudos button if you did, or please PM/DM me if you found it disruptive and I will take care of it. -BB
Manhattan GMAT Online Marketing Associate
Joined: 14 Nov 2013
Posts: 272
Own Kudos [?]: 51 [1]
Given Kudos: 0
Send PM
GMAT Critical Reasoning Problems: Arguments That Tell You Why [#permalink]
1
Bookmarks
Expert Reply
FROM Manhattan GMAT Blog: GMAT Critical Reasoning Problems: Arguments That Tell You Why

There are really only a dozen different Critical Reasoning problems in the Official Guide to the GMAT. The test writers recycle the same basic argument structures over and over, and they use the same right answers over and over, too. Even though the topics change — an argument might be about school funding the first time you see it, and industrial efficiency the next — you can sometimes recognize the underlying structure, outsmart the test, and earn some well-deserved points on the Verbal section.

One example to be familiar with is the “tell me why” argument. When you see one of these arguments, it’ll probably be part of a Weaken the Argument or Find the Assumption Critical Reasoning problem. First, the author describes a phenomenon that he or she has observed. Then, in the conclusion, the author speculates on what caused the phenomenon.

Here are some examples:

There’s no toilet paper in the bathroom. My roommate must not have bought any.

Ever since the new toll was enacted on the interstate, I’ve seen less traffic on my way to work in the mornings. The toll must have led some drivers to start commuting by public transit rather than by car.

A deficiency of Vitamin D has been shown to contribute to illness. Adequate exposure to sunlight is necessary for the human body to produce sufficient Vitamin D. However, in Seattle, where there is little natural sunlight for much of the year, residents report rates of illness that are no higher than the national average. It’s clear that after residing for some time in a place with minimal natural sunlight, the human body adapts to require a lower level of Vitamin D to avoid illness.

Each of these “tell me why” arguments describes an observation, then tries to explain why it happened. More interestingly, each of them has the same flaw: what if there was actually a different explanation that made just as much sense? That would hurt the author’s argument.

My roommate bought toilet paper, but forgot to take it out of the car on her way upstairs. That explains why it isn’t in the bathroom.

The toll is only in effect during rush hour. The same number of people are commuting by car, but now they’re doing it at different times of the day to avoid being charged.

People who live in Seattle often take Vitamin D supplements, so they have the same levels in their bodies on average as people who live in sunnier areas. They don’t get sick because they actually don’t have a Vitamin D deficiency at all.

When one of these arguments is part of a Weaken problem, the right answer will very often give another, better explanation, or at least suggest that there is one. That means that the right answer can include information that has nothing to do with the argument, at least on the surface! When you see an answer choice that looks “out of scope”, ask yourself whether it might be giving an alternative explanation for the facts. That would make it a good Weakener.

These arguments also often appear in Assumption problems. In these cases, the right answer may describe something that the author has assumed not to be true.

… assuming that my roommate didn’t accidentally leave the toilet paper in her car.

… assuming that people are still commuting to work at the same time of day they did previously.

… assuming that people in Seattle don’t use Vitamin D supplements.

The arguments themselves don’t say anything about supplements, or about the time of day, or about my roommate’s car, so aren’t these assumptions out of scope? No. By creating a “tell me why” argument, the author is always assuming that the other possible explanations aren’t correct.

Start looking out for “tell me why” arguments on Critical Reasoning, and anticipate how the right answers might look! For practice, try out these Critical Reasoning problems from The Official Guide to the GMAT, 2016: 48, 69, 90, and 121.


Chelsey Cooley is a Manhattan Prep instructor based in Seattle, Washington. Chelsey always followed her heart when it came to her education. Luckily, her heart led her straight to the perfect background for GMAT and GRE teaching: she has undergraduate degrees in mathematics and history, a master’s degree in linguistics, a 790 on the GMAT, and a perfect 170/170 on the GRE. Check out Chelsey’s upcoming GMAT prep offerings here.

The post GMAT Critical Reasoning Problems: Arguments That Tell You Why appeared first on GMAT.
This Blog post was imported into the forum automatically. We hope you found it helpful. Please use the Kudos button if you did, or please PM/DM me if you found it disruptive and I will take care of it. -BB
Manhattan GMAT Online Marketing Associate
Joined: 14 Nov 2013
Posts: 272
Own Kudos [?]: 51 [0]
Given Kudos: 0
Send PM
Three things to love about GMAT Roman numeral problems [#permalink]
Expert Reply
FROM Manhattan GMAT Blog: Three things to love about GMAT Roman numeral problems

I. Roman numeral Quant problems aren’t a whole lot of fun.

II. A lot of my students choose to skip them entirely, which is much smarter than wasting five minutes wondering what to do!

III. However, it’s possible to turn this rare and tricky problem type into an opportunity.

Read on, and learn why many GMAT high-scorers love Roman numeral problems.

Reason 1: They’re always easier than they look.
Roman numeral problems are a little bit like Data Sufficiency problems. The difficulty usually doesn’t come from the math itself; it comes from complicated logic and deliberately obscure writing. You should always start a tricky-looking Data Sufficiency problem by translating and simplifying the problem, and you can do the same on Roman numeral problems.

Here’s a problem from the GMAC’s GMATPrep software.



 

At first glance, this problem is a mess. If you simplify the problem before you begin approaching it, though, you’ll discover a much easier problem underlying it. Start with the question stem, just like in Data Sufficiency:



 

 

 

 

 

Then, simplify the statements, starting with the first:



 

 

 

 

 

 

 

 

 

The first statement actually simplifies to exactly what you’re given in the question itself, so it must be true. As an exercise, simplify the second statement in the same way. It simplifies to ad > bc, which isn’t true.

At this point, you know that the right answer must include I, and it can’t include II. You could move on and simplify the third, much more complex statement, but do you really have to?

Reason 2: There’s partial credit.
Well, not technically. But take another look at those answer choices.



 

 

 

 

 

 

 

 

 

You can actually eliminate all but two of them. Only (B) and (E) meet the criteria of including Roman numeral I, but not including Roman numeral II. If you’re quick enough with algebra, you could get to this point 1 minute into the problem — and a 50/50 guess after 1 minute beats a definitive answer after 3 minutes, when it comes to maximizing your final score. Since Roman numeral problems often have one or two statements that are much simpler than the others, they represent a fantastic opportunity to take a good guess quickly.

But if you’ve got plenty of time to solve a Roman numeral problem all the way through, what do you do?

Reason 3: Testing easy cases works really well.
I mentioned above that the difficulty of Roman Numeral problems doesn’t usually come from the math. When you test cases, you’ll rarely be tripped up by tough arithmetic or lengthy calculations. Plus, problems are often designed so that it’s clear what cases you should test. When they aren’t, you can sometimes get enough information by just testing a simple case at random. Try that approach with the third statement.

Since ad < bc, choose the following values:

a = 1                       b = 2

d = 1                      c = 2

Then, plug those values into statement III:



 

 

 

 

 

 

 

 

Since that isn’t true, statement III doesn’t have to be true. The correct answer to the problem is (B) I only. The thing to notice here is how straightforward it was to test that case — and how, unlike in Data Sufficiency, testing just one case sometimes gives you all the information you need.

What to do next
Roman numeral problems are rare on the GMAT. There’s no reason to spend hours studying them, since you might not see even a single one on any given Quant section. That said, they’re sometimes easier than they look! After reading this article, you have all of the basic tools you need to approach most Roman numeral problems. To test them out, try the following problems from the Official Guide to the GMAT, 2016: Problem Solving 66, 96, and 107.


Chelsey Cooley is a Manhattan Prep instructor based in Seattle, Washington. Chelsey always followed her heart when it came to her education. Luckily, her heart led her straight to the perfect background for GMAT and GRE teaching: she has undergraduate degrees in mathematics and history, a master’s degree in linguistics, a 790 on the GMAT, and a perfect 170/170 on the GRE. Check out Chelsey’s upcoming GMAT prep offerings here.

The post Three things to love about GMAT Roman numeral problems appeared first on GMAT.
This Blog post was imported into the forum automatically. We hope you found it helpful. Please use the Kudos button if you did, or please PM/DM me if you found it disruptive and I will take care of it. -BB
Manhattan GMAT Online Marketing Associate
Joined: 14 Nov 2013
Posts: 272
Own Kudos [?]: 51 [0]
Given Kudos: 0
Send PM
GMAT Grammar Weekly: FANBOYS [#permalink]
Expert Reply
FROM Manhattan GMAT Blog: GMAT Grammar Weekly: FANBOYS

Join us each week for a commonly-tested grammar factoid that will improve both your accuracy and your confidence on GMAT Sentence Correction.

Do you ever find yourself going on and on? And on? And on? Sentences can do the same. Sometimes  it works, but often it doesn’t. Let’s talk about when a GMAT sentence is a run-on (grammatically incorrect) and when it’s just very long (but grammatically correct).

A run-on sentence is any sentence that smushes two independent clauses together. Let’s take two simple clauses as an example.

Clause 1: Sara is wearing a purple shirt. Clause 2: She is wearing pink pants.

Both are independent clauses (meaning they can stand alone as their own sentence), but bring them together and trouble ensues:

Sara is wearing a purple shirt, she is wearing pink pants.

It’s not only a fashion faux pas, but also a grammatical one. So, how to fix it? FANBOYS to the rescue! Any of these seven simple conjunctions will turn a run-on sentence into a compound, and correct, sentence.

F             For

            And

N             Nor

B             But

O             Or

Y             Yet

S              So

Try it: Sara is wearing a purple shirt, yet she is wearing pink pants. Now we’re talking!

Now you know what FANBOYS are, but it’s time to talk GMAT. GMAT sentences are often incredibly complex, so we have to break down the complexity. Let’s practice on a much more complicated sentence:

After watching the documentary, Joe and Aaron, who had been friends since elementary school, stayed up most of the night talking about the pros and cons of the country’s leadership, which had been the subject of much criticism throughout the movie, they decided that many of the conclusions the documentary made were fallacious.

Take a moment to break this very long sentence down to it’s core. Find the main subject and verb (more on how to do this in the Sentence Correction Strategy Guide if you need more help). Start by eliminating all the modifiers:

After watching the documentary, Joe and Aaron, who had been friends since elementary school, stayed up most of the night talking about the pros and cons of the country’s leadership, which had been the subject of much criticism throughout the movie, they decided that many of the conclusions the documentary made were fallacious.

Already, we have a much simpler sentence. Keep breaking it down until you see the main subject-verb: “Joe and Aaron stayed up.” But we have a problem. The last clause is “They decided.” Both of those clauses are independent, so we should look for one of the FANBOYS.  Go ahead and look, but you won’t find. We’ve come across the fatal error of this sentence. Insert one of the FANBOYS, and you’ve got a sentence:

After watching the documentary, Joe and Aaron, who had been friends since elementary school, stayed up most of the night talking about the pros and cons of the country’s leadership, which had been the subject of much criticism throughout the movie, AND they decided that many of the conclusions the documentary made were fallacious.

So Grammar tip of the week in a nutshell: FANBOYS are used to turn run-on sentences into compound sentences. 



Emily Madan is a Manhattan Prep instructor based in Philadelphia. Having scored in the 99th percentile of the GMAT (770) and LSAT (177), Emily is committed to helping others achieve their full potential. In the classroom, she loves bringing concepts to life and her greatest thrill is that moment when a complex topic suddenly becomes clear. Check out Emily’s upcoming GMAT courses here!

The post GMAT Grammar Weekly: FANBOYS appeared first on GMAT.
This Blog post was imported into the forum automatically. We hope you found it helpful. Please use the Kudos button if you did, or please PM/DM me if you found it disruptive and I will take care of it. -BB
Manhattan GMAT Online Marketing Associate
Joined: 14 Nov 2013
Posts: 272
Own Kudos [?]: 51 [1]
Given Kudos: 0
Send PM
Here’s How to do GMAT Unit Conversions Like a Pro [#permalink]
1
Bookmarks
Expert Reply
FROM Manhattan GMAT Blog: Here’s How to do GMAT Unit Conversions Like a Pro

Sometimes the whole point of a specific GMAT problem is to convert between miles and kilometers, or meters and centimeters. In other problems, you’ll need to do a unit conversion as part of a longer solution. It’s easy to mess up unit conversions, and the GMAT writers know this — they include them on the test in order to test your level of organization and your ability to double-check your work. Here’s how to add fast unit conversions to your repertoire of skills.

  • Write the units down
The easiest mistake to make is simply forgetting what units you’re working with. If the problem asks for a number of cents, but you calculate a number of dollars, you’ll be off by a factor of 100 even if you do all of the math correctly. If there’s any chance that units will come into play in a problem, write them out at every step.

[*] Treat units like variables[/list]
Units can be multiplied, divided, and canceled out, exactly like variables. (Remember that ‘per’ always translates to division.) Suppose you’re converting 1400 crowns to rupees, and you know that the exchange rate is 0.4 crowns per rupee. Do you multiply 1400 by 0.4, or do you divide? Check by trying to cancel out the units:

Wrong:




 





 

 

 

Right:








Since division causes the units to correctly simplify to rupees, division is correct. Think about which units will need to cancel, and arrange them so that they do.

[*] One step at a time[/list]
With complex unit conversions, don’t skip steps. Convert a single unit at a time. In this example, we’ll convert 15 meters per second to kilometers per hour, by first converting seconds to minutes.



All that’s left is to do the arithmetic, by calculating (15 x 60 x 60)/1000. The result is 54 kilometers per hour.

[*] Sanity check[/list]
Do a basic sanity check each time you finish a unit conversion using familiar units. If you’re converting two hundred dollars to cents, should the result be greater than 200 or less than 200? If you’re converting 45 seconds to hours, should the result be greater than 45 or less than 45? It’s easy to mix up multiplication and division, and quickly checking to make sure your result is sensible can help you avoid this.

[*] Practice with Google[/list]
Type “15 meters per second in kilometers per hour” into a Google search box. Google Calculator automatically handles many unit conversions, including complex ones. That means that you can use it to drill this skill after reading this article! Make up a few complex unit conversions and simplify them on paper, exactly as you would if you saw them on the GMAT. Then, use Google to check your answers.

Half an hour of work, right now, will make you much quicker and more confident at unit conversions. While they don’t appear in every Quant problem, the GMAT test writers love to throw them in at the end of a tricky problem, in the hopes that unprepared test takers will make an easy mistake. Make them part of your GMAT toolkit, and outsmart the test! 


Chelsey Cooley is a Manhattan Prep instructor based in Seattle, Washington. Chelsey always followed her heart when it came to her education. Luckily, her heart led her straight to the perfect background for GMAT and GRE teaching: she has undergraduate degrees in mathematics and history, a master’s degree in linguistics, a 790 on the GMAT, and a perfect 170/170 on the GRE. Check out Chelsey’s upcoming GMAT prep offerings here.

The post Here’s How to do GMAT Unit Conversions Like a Pro appeared first on GMAT.
This Blog post was imported into the forum automatically. We hope you found it helpful. Please use the Kudos button if you did, or please PM/DM me if you found it disruptive and I will take care of it. -BB
Manhattan GMAT Online Marketing Associate
Joined: 14 Nov 2013
Posts: 272
Own Kudos [?]: 51 [0]
Given Kudos: 0
Send PM
GMAT Critical Reasoning Problems: Benefit/Drawback Arguments [#permalink]
Expert Reply
FROM Manhattan GMAT Blog: GMAT Critical Reasoning Problems: Benefit/Drawback Arguments

Here are a few benefit/drawback arguments:

1. Inexperienced workers are willing to take jobs with lower pay and fewer benefits. So, FantasyCorp should exclusively hire new workers with little experience.

2. Regular exercise reduces the risk of heart disease. To protect my heart, I exercise for three hours every day.

3. Certain chemical pesticides leave a residue on the leaves of plants treated with them. This pesticide residue is known to kill caterpillars that harmlessly feed on fallen leaves, ensuring that they never mature into butterflies. If farmers in GMATopia continue using chemical pesticides on their crops rather than alternative methods of pest control, they will inevitably decimate the butterfly population in the area.

In the first two arguments, the author promotes a plan. In the third argument, the author predicts negative consequences. In Critical Reasoning arguments, the author is never 100% correct: if he or she is promoting the plan, then there’s probably a hidden drawback. If he or she disagrees with the plan, there may be a hidden benefit, or perhaps one of the author’s claims about drawbacks will turn out to be irrelevant.

1. One drawback to hiring inexperienced workers is that they’re likely to do their jobs poorly.

2. The second argument is a little more complex. Here are two possible drawbacks: think of them as two possible answers to a Weaken the Argument question.

Wrong: Exercising for three hours a day is time-consuming, making it difficult to keep a job or pursue other hobbies.

Right: Exercising for more than ten hours each week has been shown to cause lasting damage to heart muscle.

Only the second option is an acceptable GMAT answer.

On the GMAT, you have to address the conclusion exactly as it’s written. It says, specifically, that the author exercises to protect his heart. A drawback will have to show that exercising that much might not actually keep his heart healthy.

3. In the third argument, the author predicts that the farmers’ use of pesticides will have a specific negative consequence. To prove him wrong, you’d have to show that the butterfly population might not suffer: either by finding a hidden benefit that outweighs the drawbacks, or by proving his claims about drawbacks incorrect.

Wrong: Butterflies have been shown to harm many common crops.

Right: Most chemical pesticides, including the ones commonly used in GMATopia, are actually harmless to caterpillars.

The conclusion doesn’t say “we shouldn’t decimate the butterfly population,” even though the author probably does believe that! The conclusion claims that “using chemical pesticides will decimate the butterfly population.” Only the second option given above actually addresses that claim.

Now, imagine seeing these arguments in a Strengthen the Argument problem. Do the reverse of what we’ve already done: add an additional benefit or drawback, or address the relevance of one that’s already given. The following statements all strengthen these cost/benefit arguments.

1. Inexperienced workers are generally more careful with their work, and have fewer bad habits.

A cost-benefit analysis has shown that the cost of training and supervising inexperienced workers is significantly less than the loss accrued due to paying higher salaries to more senior workers.

2. The heart-protective effects of exercise increase in direct proportion to the amount of exercise performed.

 Regular exercise also prevents the normal deleterious effects of aging on heart tissue.

3. The type of chemical pesticide commonly used in GMATopia kills butterflies that happen to alight on crops that have been sprayed with it, as well as killing caterpillars that feed on these crops.

Farmers generally use harmful chemical pesticides on their crops in the springtime, which is the main feeding season for young caterpillars.

Work out how each of these facts would successfully strengthen the arguments above. Can you also think of some incorrect “strengtheners” that actually miss the point, similar to the incorrect “weakeners” provided earlier? Can you think of some assumptions that are being made in these arguments? (Think about benefits and drawbacks when you identify assumptions — if the author believes the plan is good, he or she is assuming that it lacks major drawbacks.) If you can, then you understand the benefit/drawback argument type.

As you practice with the Critical Reasoning Strategy Guide, or the Official Guide to the GMAT, look for this type of argument. When you notice one, remember to identify the specific conclusion, steer clear of tempting “real world” answers, and think about adding, removing, supporting, or disagreeing with benefits and drawbacks. The next time you see one of these arguments, predict some possible right answers before you check the answer choices. You may surprise yourself with your accuracy!


Chelsey Cooley is a Manhattan Prep instructor based in Seattle, Washington. Chelsey always followed her heart when it came to her education. Luckily, her heart led her straight to the perfect background for GMAT and GRE teaching: she has undergraduate degrees in mathematics and history, a master’s degree in linguistics, a 790 on the GMAT, and a perfect 170/170 on the GRE. Check out Chelsey’s upcoming GMAT prep offerings here.

 

The post GMAT Critical Reasoning Problems: Benefit/Drawback Arguments appeared first on GMAT.
This Blog post was imported into the forum automatically. We hope you found it helpful. Please use the Kudos button if you did, or please PM/DM me if you found it disruptive and I will take care of it. -BB
Manhattan GMAT Online Marketing Associate
Joined: 14 Nov 2013
Posts: 272
Own Kudos [?]: 51 [0]
Given Kudos: 0
Send PM
How to Master Every GMAT Critical Reasoning Question Type [#permalink]
Expert Reply
FROM Manhattan GMAT Blog: How to Master Every GMAT Critical Reasoning Question Type

Has GMAT Critical Reasoning been driving you crazy? Do you keep getting tangled up in arguments, agonizing back and forth between answers, or picking an answer confidently only to find that you fell straight into a trap? This article is here to save you. ☺️

It’s going to take some work, but if you follow these steps, you’ll see your CR performance improve significantly. Ready? Let’s do this!

Open up an Excel spreadsheet (Seriously!)…

Now, set up this template on the first worksheet:


Label this first worksheet your Key.

Are you wondering why I didn’t just give you an Excel file with this template? I specifically want you to type out each step and think about what it means. You’re about to apply this analysis to the different question types; you’ll do so more effectively if you have a solid understanding of what you’re trying to accomplish.

If you aren’t using Manhattan Prep’s CR book, then read my blog post on our CR process. It explains the four steps listed above.

Now, create new worksheets in the spreadsheet.

You’re going to need a total of 10 worksheets, including the first worksheet that you already created above. Copy that worksheet 9 times.

Now label those other 9 questions in this way (and in this order!):

  • Assump (for Find the Assumption)
  • Str (for Strengthen the Argument)
  • Weak (for Weaken the Argument)
  • Infer (for Inference)
  • Discrep (for Find the Discrepancy)
  • Role (for Describe the Role) (could also call this BF for Boldface)
  • Eval (for Evaluate the Argument)
  • Flaw (for Find the Flaw)
  • DA (for Describe the Argument)
I put these in this order because this is the rough order of importance based on the frequency with which these are tested. The three most common types are Assumption, Strengthen, and Weaken; you will most likely see more than one of each of these. After that, the next four (Infer, Discrepancy, Role, and Evaluate) are about equally common; you will most likely see just one of each. The Flaw and

Describe the Argument questions are the least frequently seen; you may or may not see one of these.

Start filling out your templates!

Each question type now has its own worksheet with the template. Your task is to start replacing the template with your answers to those questions for each question type. Get going!

Is that all?

Okay. I’ll give you a little more. ☺️

Here’s what I have in my template for the Describe the Role question type. (I chose a medium-frequency one because I really want you to put the three most common ones in your own words. You’re going to use those the most.)



Ready? Set? Go!

Okay, you’ve got your instructions—now go make it happen! This template will help you to know exactly what to ask yourself and what to examine while you’re working through any CR problem. Want to test out your templates? Take a look at this Master Resource CR article. It contains links to articles on every type of CR problem with the exception of Describe the Argument (these are really pretty rare). Each article gives you a problem to try (from the free resources in GMAT Prep) and then analyzes it thoroughly using this same 4-step process.

Happy Studying!

Live in L.A.? I’m teaching a GMAT Complete Course in West Hollywood starting February 2nd. Anyone is welcome to attend the first class for free. I hope to see you there!


Stacey Koprince is a Manhattan Prep instructor based in Montreal, Canada and Los Angeles, California.
Stacey has been teaching the GMAT, GRE, and LSAT  for more than 15 years and is one of the most well-known instructors in the industry. Stacey loves to teach and is absolutely fascinated by standardized tests. Check out Stacey’s upcoming GMAT courses here

The post How to Master Every GMAT Critical Reasoning Question Type appeared first on GMAT.
This Blog post was imported into the forum automatically. We hope you found it helpful. Please use the Kudos button if you did, or please PM/DM me if you found it disruptive and I will take care of it. -BB
Manhattan GMAT Online Marketing Associate
Joined: 14 Nov 2013
Posts: 272
Own Kudos [?]: 51 [0]
Given Kudos: 0
Send PM
Just Started Studying for the GMAT? Here’s Where to Begin [#permalink]
Expert Reply
FROM Manhattan GMAT Blog: Just Started Studying for the GMAT? Here’s Where to Begin

When you first look at the resources available to get you through the GMAT, it’s easy to get overwhelmed. Should you read through all the strategy guides? Complete every Official Guide problem you can find? Sign up for every workshop? Let’s breakdown your options and take this step by step.

Step 1: Assess your comfort with math

You may already know you need some extra help, but if you’re unsure, start with the basic math diagnostic. If you stumble through it, it’s time to build up you foundation. You can work through our foundations of math strategy guide, attend the two foundations workshops, or find review sheets for each of the topics tested online. You should be comfortable with manipulating different kinds of numbers, including fractions, percents, and square roots. You do not, however, have to be completely confident in your arithmetic before you start the next steps.

Step 2: Assess your comfort with the GMAT

If you either score in the midrange of the basic math diagnostic, or have started working through the foundations, it’s time to see how you do on a full test. Take a GMAT Practice Test. You can find a free one in the list of free resources on our site, labeled practice CAT (Computer Adaptive Test). Don’t worry about the score yet.

Instead, use the test as it’s meant: as a diagnostic. Analyze the results carefully, using this blog post series to help guide you. While you should focus more on areas of weakness at this point, you should make sure not to ignore any major topic. A course, the interact lessons, or the self study toolkit can each help keep you on track, as well as providing a schedule that will tell you what to work on.

Step 3: The heart of the preparation

Now that you’ve analyzed the exam and have taken a refresher course on math basics, it’s time to get to the heart of the preparation. Where you go now depends on your level and the way you learn. Take a moment to reflect on how you have mastered topics in the past. Did you take classes? Watch videos? Read through textbooks? If you have a strong preference for a certain learning style, that can help you decide the next steps. If not, you need to rely a bit more on your level and availability. Let’s discuss the top three studying styles.

Take a class. 

Consider this option if you like a little extra structure, learn well in study groups, or want a teacher’s expertise without the one-on-one feel of a tutor.  Each week, you’re accountable for specific homework to prepare you for the upcoming session, but if you fall behind, there’s plenty of opportunity to catch up after class. You get the benefit of an expert working you through the most challenging problems, as well as the chance to learn from the mistakes and breakthroughs of students in situations similar to yours. Check out classes near you.

Private tutoring.

Think about private tutoring if you need a more flexible schedule, have specific areas/issues you want to work on, or get the most benefit from a one-on-one with an expert. The study schedule will be tailored to your needs and availability, and you’ll have the opportunity to explore different topics in as much or as little depth as you need. The sessions will require a lot of work since each moment will be focused on improving your specific performance, but that work will pay off with a large benefit.

Self-studying.

Choose this option if you’ve had a lot of success with independent study in the past. However, do not be fooled into thinking that completing as many Official Guide problems as possible is the same as self-studying. By themselves, OG problems are not particularly helpful in getting ready for the test. Instead invest in a set of strategy guides, our self-study tool-kit, or the interact lessons. You’ll learn not only the content, but also the strategies that will result in significant score improvement.

Pick your preference, and get studying!



Emily Madan is a Manhattan Prep instructor based in Philadelphia. Having scored in the 99th percentile of the GMAT (770) and LSAT (177), Emily is committed to helping others achieve their full potential. In the classroom, she loves bringing concepts to life and her greatest thrill is that moment when a complex topic suddenly becomes clear. Check out Emily’s upcoming GMAT courses here. Your first class is always free!

The post Just Started Studying for the GMAT? Here’s Where to Begin appeared first on GMAT.
This Blog post was imported into the forum automatically. We hope you found it helpful. Please use the Kudos button if you did, or please PM/DM me if you found it disruptive and I will take care of it. -BB
Manhattan GMAT Online Marketing Associate
Joined: 14 Nov 2013
Posts: 272
Own Kudos [?]: 51 [0]
Given Kudos: 0
Send PM
Here’s What to Do When You Don’t Know What to Do on the GMAT [#permalink]
Expert Reply
FROM Manhattan GMAT Blog: Here’s What to Do When You Don’t Know What to Do on the GMAT

You’re staring at a GMAT problem that you just don’t understand. There’s a minute left on the clock. What do you do?

Are you behind on time?
If so, guess randomly and move on. Making up for lost time is much more valuable than anything I’ll describe in this article. Even if you’re ahead on time, consider moving on from the problem anyway, especially if you tend to work slowly.

Question-Type Strategies
Integrated Reasoning

Unless it’s the very last problem, or you’re way ahead on time, guess randomly and move on. Integrated Reasoning rewards strategic skipping more than any other problem type, since the problems often have multiple parts (making it much less likely that you’ll get the right answer by guessing) and the difficulty comes mainly from intense time pressure.

Problem Solving

  • Eliminate “Homers.” A Homer is any answer choice that Homer Simpson might pick. Homer is clueless, so he’d probably just look at the numbers in the problem, look at the answer choices, and pick one that looks similar. The test writers expect this, so don’t be like Homer. If an answer choice contains a lot of the numbers from the problem, or if it’s a simple combination of those numbers (like a sum or a product), don’t pick it.
  • If there are two variables in the problem, eliminate singletons. Suppose that a problem states that the sum of Archie’s and Betty’s ages is 38. The test writers will probably include both Archie’s actual age and Betty’s actual age among the answer choices, just because it’s easy to slip up and pick the wrong one. The right answer, and one of the wrong answers, will sum to 38. Eliminate any answers that aren’t part of such a pair.
  • Benchmark. If you understand the problem a little, but you don’t know how to do the math, check the answer choices. Are some of them greater than 1, and some less than 1? Are some of them very large, and others very small? If so, guess which category the right answer would belong to.  When in doubt, pick an answer choice that looks complex over one that looks simple.
Data Sufficiency

  • Don’t forget that you can work with just the easier statement. This will allow you to eliminate either 2 or 3 answer choices.
  • If the two statements look almost identical, guess A or B. The test writers are probably trying to see if you can identify a small but crucial difference between the statements.
  • In general, avoid guessing C. On the one hand, C is the right answer 20% of the time, just like any other answer choice. On the other hand, a lot of logically complex problems are designed to trick you into choosing C. If C feels right, but you’re not really sure why, it’s probably a trap.
Sentence Correction

  • Mentally cross off modifiers to find the core of the sentence. If you’re really struggling to understand the sentence, there’s probably something wrong with the core.
  • Find a single, easy, grammatical split to work with. Hard problems often have a single, subtle pronoun or subject/verb agreement split that’ll let you eliminate 2 or 3 answer choices.
  • Never guess an answer choice if you know there’s a grammatical error in it, even if it sounds the best overall. The right answer will never have any errors.
  • If you’re out of ideas, it’s okay to go with what sounds right. That’s the great thing about Sentence Correction — on a Quant problem, you can get completely stuck and have no idea where to start. On Sentence Correction, you always have your instincts as an English speaker!
Reading Comprehension

  • Look at the first few words of every answer choice if you’re guessing the answer to a general Reading Comprehension question, such as a Main Idea question. They’ll usually include a term such as explain or argue. Eliminate any answer choices that don’t seem to describe the passage you read. Don’t guess an answer choice containing argue or advocate unless the author is obviously doing that. Just because the author seems to have a slight preference or voices a positive or negative about something, you can’t necessarily describe the point of the passage as arguing.
  • Avoid guessing an answer that contains too many keywords from the passage. It’s probably a trap.
  • If there’s anything wrong with an answer choice, it’s wrong. Wrong answers will often have a single incorrect word or phrase, but otherwise look great — don’t pick an answer that looks mostly right if you can identify anything wrong with it.  
Critical Reasoning:

  • Pick the most boring, wishy-washiest answer if it’s a Draw a Conclusion or Find the Assumption problem. The right answers to these problem types rarely consist of bold statements.
  • Don’t pick a guess that contains strong language, such as “all” or “every” or “never”. These are sometimes right, but they’re often traps.
  • Don’t pick a guess that makes a comparison, such as “GMATopia exports more books than any other country.” These answer choices are sometimes right, but they’re also often inserted to trick you.
These aren’t the only ways to make a quick strategic guess, but they’re a few of the best ones.

As you practice, you’ll observe other patterns in which answers tend to be right or wrong. Which wrong answer choices you’re tempted by, or which answers tend to be correct on problems that you struggle with, will also depend on your own habits as a test-taker. Use this list as a jumping-off point, but whenever you make a successful guess — or whenever you notice a clever way to guess while reviewing a problem — write down what you’ve discovered, and use it on test day. Guessing is necessary for everyone, and guessing is a skill you can learn — that’s why we teach good guessing skills in our 9-week GMAT Complete Courses, alongside lots of real math and language content. Improve your guessing abilities now to avoid trouble when you take the official GMAT. 

Chelsey Cooley
is a Manhattan Prep instructor based in Seattle, Washington.
 Chelsey always followed her heart when it came to her education. Luckily, her heart led her straight to the perfect background for GMAT and GRE teaching: she has undergraduate degrees in mathematics and history, a master’s degree in linguistics, a 790 on the GMAT, and a perfect 170/170 on the GRE. Check out Chelsey’s upcoming GMAT prep offerings here.

 

The post Here’s What to Do When You Don’t Know What to Do on the GMAT appeared first on GMAT.
This Blog post was imported into the forum automatically. We hope you found it helpful. Please use the Kudos button if you did, or please PM/DM me if you found it disruptive and I will take care of it. -BB
Manhattan GMAT Online Marketing Associate
Joined: 14 Nov 2013
Posts: 272
Own Kudos [?]: 51 [0]
Given Kudos: 0
Send PM
GMAT Sentence Correction: What can the underline tell you? [#permalink]
Expert Reply
FROM Manhattan GMAT Blog: GMAT Sentence Correction: What can the underline tell you?

I ran across the GMAT problem below when I was reviewing a GMATPrep® test that I took a while back, and as soon as I saw it, I knew I needed to share it with you. There are some really intriguing aspects to this one.

[Want to learn even more about GMAT Sentence Correction? Check out our famous Manhattan Prep GMAT Sentence Correction Strategy Guide.]

First, try it out. (Note: in the solution, I’m going to discuss aspects of our SC Process; if you haven’t learned it already, go read about it right now, then come back and try this problem.)

* “Analyzing campaign expenditures, the media has had as a focus the high costs and low ethics of campaign finance, but they have generally overlooked the cost of actually administering elections, which includes facilities, transport, printing, staffing, and technology.

“(A) Analyzing campaign expenditures, the media has had as a focus”

“(B) Analyses of campaign expenditures by the media has been focused on”

“(C) In analyzing campaign expenditures, the media have focused on”

“(D) Media analyses of campaign expenditures have had as a focus”

“(E) In their analysis of campaign expenditures, the media has been focusing on”

First Glance

The comma in the middle of the underline jumped out at me. There’s probably some kind of modifier—I’m going to keep an eye out for that when I read the full sentence.

Next, glance down the beginning of each choice.

There are some pretty significant differences. I’m not sure what they mean yet, but this is reinforcing the idea that I need to get a handle on the core sentence vs. the modifiers.

I also noticed how very long the non-underlined portion is at the end—and that immediately made me suspicious. What are they trying to hide from me?

Next, I read the original sentence and realized that I’ve got an opening modifier.

It sounds a little funny to me. I’m not sure why.

Then I thought back to that really long non-underline.

Often, this means that there’s some word or sequence of words in the non-underlined part that has to connect with something in the underlined part. If I can figure out what that is, I can cross off some wrong answers.

The best way to figure out the possibilities? Scan the answer choices vertically to find the differences.

Boom. There’s a difference between has and have: singular vs. plural. Now I need to choose: do I move forward with this or go back to that opening modifier?

I’m going to stick with this; I like subject-verb agreement better. Okay, so the subject is…oh. They actually change the sentence structure in some of the answers! The subject is not always media. Hmm. I’m going to look first only at choices that use media as the subject (which the original sentence does).

Answers (A), (C), and (E) fit the bill. (A) and (E) use media has. But (C) uses media have. Is media singular or plural?

It depends! I’ve heard people use it both ways. So how am I supposed to be able to tell?

Whenever they use a word that could go both ways (media, data, etc.), they will always give me a clue in the non-underlined portion of the sentence. I just have to find it.

Scanning, scanning…there it is!

“they have generally overlooked the cost of actually administering elections”

The plural pronoun they is referring back to the subject,media, so I have to make the earlier part of the sentence match: media is plural. Answers (A) and (E) are both incorrect.

What about (B) and (D)? They switch up the subject there:

“(B) Analyses of campaign expenditures by the media has been focused on”

“(D) Media analyses of campaign expenditures have had as a focus”

In both cases, the subject is the plural analyses. Answer (B) pairs that subject with the singular has. Nope.

Okay, so on subject-verb agreement alone, I’m down to (C) and (D). I’m feeling pretty good.

Let’s compare the final two:

“(C) In analyzing campaign expenditures, the media have focused on”

“(D) Media analyses of campaign expenditures have had as a focus”

The final distinction here is pretty tricky. If you narrowed down to these two and then had to guess, you should still feel good about this problem.

If I were discussing my career, I could say:

In my career, I have had as a focus the GMAT.

I could also say:

In my career, I have focused on the GMAT.

The second construction is the more idiomatic construction. In general, don’t say that something “had as a focus” XYZ (or had XYZ as a focus) when you can just say something “focused on” XYZ.

Also note the implications for the second part of the sentence:

“(C) In analyzing campaign expenditures, the media have focused on…”

“(D) Media analyses of campaign expenditures have had as a focus…”

“…but they have generally overlooked the cost of actually administering elections”

In answer (C), the subject is media, so the second half of the sentence says that the media overlooked certain costs.

In answer (D), the subject is analyses, so the second half of the sentence says that the analyses overlooked certain costs.

The analyses are not animate—they can’t actually think for themselves. I could say that your analysis fails to address certain topics, but I would say that you overlooked those topics.

I will admit that I have heard people say something similar to “your analysis overlooks blah blahblah,” but that’s generally because people don’t enjoy directly criticizing others (at least…most of us don’t!). It’s more confrontational to say that you overlooked it—even though that is more correct.

The correct answer is (C).

I also want to point out a subtle issue at the beginning of answer choice (A). We didn’t need to use this to eliminate, but it’s still something you should know.

The opening modifier in (A) is a comma –ing modifier. These kinds of modifiers apply to the subject and verb of the core sentence. In addition, comma –ing modifiers imply a certain sequence to the events presented. Take a look at these (correct) examples:

Running as fast as she could, she barely caught the bus.

She ran as fast as she could, barely catching the bus.

The event that is presented first in the sentence is also the one that happens first in the sequence of events. She ran as fast as she could; as a result, she caught the bus.

You would not say:

She barely caught the bus, running as fast as she could.

This implies that she caught the bus, and as a result, she ran as fast as she could. That’s illogical! The sequence is out of order.

So in the original sentence of our GMATPrep problem, we have:

“Analyzing campaign expenditures, the media has had as a focus…”

First, they analyzed campaign expenditures…and then, as a result, they had a particular focus?

No! If anything, first they had a focus and then they used that focus to drive their analysis. Or you could say that these things were simultaneous; they’re really the same event.

Answer (C) fixes this by turning that opening modifier into a prepositional phrase:

“(C) In analyzing campaign expenditures, the media have focused on…”

In this kind of construction, think of in as a synonym for while or in the process of. In the process of analyzing this stuff, they focused on blah blah blah.

Key Takeaways for a long, non-underlined block of text
(1) Keep an eye out for this characteristic. They’re probably trying to hide something from you. Compare the answers (vertically!) to spot the differences. Then, think about what you would need to know to make the call for those various differences.

(2) If they give you a “two-way” noun (one that can be singular or plural), then they have to give you a clue to tell you which way to go, and that clue is going to be in the non-underlined portion of the sentence. It could be a pronoun (as in this problem) or a verb. Go find it!

(3) It’s really important to be able to find the core sentence. Want more practice with that skill? Check out this series.

Want to learn more about GMAT Sentence Correction? Check out our famous Manhattan Prep GMAT Sentence Correction Strategy Guide.

* GMATPrep® questions courtesy of the Graduate Management Admissions Council. Usage of this question does not imply endorsement by GMAC.


Stacey Koprince is a Manhattan Prep instructor based in Montreal, Canada and Los Angeles, California.
Stacey has been teaching the GMAT, GRE, and LSAT  for more than 15 years and is one of the most well-known instructors in the industry. Stacey loves to teach and is absolutely fascinated by standardized tests. Check out Stacey’s upcoming GMAT courses here

The post GMAT Sentence Correction: What can the underline tell you? appeared first on GMAT.
This Blog post was imported into the forum automatically. We hope you found it helpful. Please use the Kudos button if you did, or please PM/DM me if you found it disruptive and I will take care of it. -BB
Manhattan GMAT Online Marketing Associate
Joined: 14 Nov 2013
Posts: 272
Own Kudos [?]: 51 [0]
Given Kudos: 0
Send PM
MBA Admissions Myths Destroyed: My Supervisor Graduated from HBS—He Kn [#permalink]
Expert Reply
FROM Manhattan GMAT Blog: MBA Admissions Myths Destroyed: My Supervisor Graduated from HBS—He Knows!
The Following post has been brought to you by our friends at mbaMission.


What have you been told about applying to business school? With the advent of chat rooms, blogs, and forums, armchair “experts” often unintentionally propagate MBA admissions myths, which can linger and undermine an applicant’s confidence. Some applicants are led to believe that schools want a specific “type” of candidate and expect certain GMAT scores and GPAs, for example. Others are led to believe that they need to know alumni from their target schools and/or get a letter of reference from the CEO of their firm in order to get in. In this weekly series, mbaMission debunks these and other myths and strives to take the anxiety out of the MBA admissions process.

We at mbaMission know of a now 70-year-old man who graduated from a virtually unknown Canadian undergraduate school in 1963 and who, with no work experience at all, applied to Harvard Business School (HBS), Wharton, and the Stanford Graduate School of Business (GSB), earning acceptance at all three (though the GSB deferred his entry for one year so he could earn a little more experience first). He ultimately studied at HBS and now runs a small grain-trading business. You could not meet a nicer man, and although he is certainly wise in many respects, one thing he knows nothing about is MBA admissions. “I attended so long ago, things must have changed since then,” he says. “I did not have any work experience at all. I had studied four years of commerce, and that was it!”

Why are we telling you this? Many candidates each year tell us that their bosses, who applied to business school during far different times, have given them “sage” advice about applying and that they feel they should follow it—after all, what worked for their boss in 1966, 1976, 1986, or even 1996 must still be applicable today, right? Wrong.

For a long time, the MBA was actually not all that desirable a degree, so admissions was not quite so competitive. To give you an idea of the MBA’s relative popularity, Duke University (Fuqua) did not even start its MBA program until 1970, but its law school was founded in 1868. Yale University was founded in 1701, but it did not have an MBA program until 1976. So, the MBA is a relatively new degree that has only recently (in the late 1990s) reached its current level of popularity and prestige.

What does all of this mean with regard to your boss’s advice? Although your supervisor may have gotten into one of your target schools, he or she likely did so years ago and therefore may not have had to contend with the steep competition you now face. Your boss may also not know anything about what the admissions process is like today and could be—however inadvertently—leading you astray. If your supervisor starts any bit of his or her well-intended advice with the phrase “when I applied,” you should view the coming declaration with tremendous caution.

mbaMission is the leader in MBA admissions consulting with a full-time and comprehensively trained staff of consultants, all with profound communications and MBA experience. mbaMission has helped thousands of candidates fulfill their dream of attending prominent MBA programs around the world. Take your first step toward a more successful MBA application experience with a free 30-minute consultation with one of mbaMission’s senior consultants. Sign up today at www.mbamission.com/manhattangmat.

The post MBA Admissions Myths Destroyed: My Supervisor Graduated from HBS—He Knows! appeared first on GMAT.
This Blog post was imported into the forum automatically. We hope you found it helpful. Please use the Kudos button if you did, or please PM/DM me if you found it disruptive and I will take care of it. -BB
Manhattan GMAT Online Marketing Associate
Joined: 14 Nov 2013
Posts: 272
Own Kudos [?]: 51 [0]
Given Kudos: 0
Send PM
The Top 6 GMAT Quant Mistakes That You Don’t know You’re Making [#permalink]
Expert Reply
FROM Manhattan GMAT Blog: The Top 6 GMAT Quant Mistakes That You Don’t know You’re Making

Did you know that you can attend the first session of any of our online or in-person GMAT courses absolutely free? We’re not kidding! Check out our upcoming courses here.


Sometimes, as you solve a GMAT Problem Solving problem, everything seems to go smoothly. You get an answer that matches one of the choices perfectly, so you select it and move on to the next problem. But much later, when you’re reviewing the problem, you realize that you picked the wrong answer entirely. Why does this happen, and how can you stop it?

According to data from GMAT Navigator, our online platform that lets students record their answers to GMAT practice problems (click here for the premium version or here for the basic version), there are certain Problem Solving problems that most students feel very confident about. Very few students guess on these problems. However, many students get these problems wrong, despite their high level of confidence. Here’s the list of mistakes these students make most frequently.

  • Off by 1: When a problem asks for a largest or smallest possible value, or when it includes inequalities, double-check your work before answering. The answer choices are often only 1 apart, and it’s easy to choose an answer that’s 1 too small or too large. For instance, what is the smallest integer that’s greater than 41/4? If you quickly divide 41 by 4 and take the integer part of the answer, 10, you’ll be off by
  • Off by 100: Be careful with percentage problems, especially ones with answer choices that are similar except for the number of decimal places (0.89, 8.9, 89, 890, etc.) If you miss the word ‘percent’ somewhere in a problem, or if you forget to multiply a decimal by 100 to convert it to a percent, your answer will be off by a factor of 100. Check out our guide to approaching percent problems if you make this mistake.
  • Bad unit conversion: It’s easy to make a mistake when converting between kilometers and miles, dollars and cents, or grams and kilograms. The worst culprits are problems that ask you to convert between units you may not be familiar with, or even made-up units, since your instincts won’t tell you that your answer is wrong. Always write out the entire unit conversion and check your work carefully. It’s more time-consuming, but it’ll ensure you never multiply when you should divide, or accidentally convert the wrong value. Read our guide to fast and safe unit conversions to get a handle on this issue.
  • Missing the last step: You get all the way through the problem, doing all of the math correctly, and solve for the value of n. Then you choose that value and move on, never noticing that the problem actually asked you to choose the value of 2n, or n2, as your answer. To avoid this, read the entire problem, and the answer choices, before you begin writing. Before you write anything else on your paper, jot down what you’re solving for and circle it.
  • Losing the units: Picture a rates & work problem that asks you to solve for the amount of time it would take two machines to complete a job when working together. Unbeknownst to you, one of the wrong answer choices is actually the rate at which the two machines are working, which is the reciprocal of the time. The answer choices don’t include units, so you have to keep track of them yourself. And be aware that the test writers might play dirty tricks with units in the text of a problem: for instance, rates are usually given in miles per hour, but I’ve seen a problem that includes a rate given in minutes per mile. Many of my students gloss right over the units, assume that the rate is written in miles per hour, and do all of the math correctly but get the wrong answer anyways. Don’t let that happen to you.
  • Percent more than/less than: 120 is 20 percent more than 100, but 100 isn’t 20 percent less than 120. (Take a moment to do the math, and figure out what number actually fits there!) You can’t switch back and forth between percent more than and percent less than, even if it seems to make the problem easier; the math just doesn’t work out. Make sure that when you translate a percent word problem into math, you’re keeping the same expressions as in the original problem.
These issues were the six most common culprits that caused students to unknowingly miss GMAT Problem Solving questions. Learn them now, and double-check your work whenever you see a suspicious problem. There’s nothing wrong with missing a problem because you haven’t learned the math yet, but you never want to miss problems that you could easily have gotten right. 

Want full access to Chelsey’s sage GMAT wisdom? Try the first class of one of her upcoming GMAT courses for absolutely free, no strings attached. 

Chelsey Cooley
is a Manhattan Prep instructor based in Seattle, Washington.
 Chelsey always followed her heart when it came to her education. Luckily, her heart led her straight to the perfect background for GMAT and GRE teaching: she has undergraduate degrees in mathematics and history, a master’s degree in linguistics, a 790 on the GMAT, and a perfect 170/170 on the GRE. Check out Chelsey’s upcoming GMAT prep offerings here.

The post The Top 6 GMAT Quant Mistakes That You Don’t know You’re Making appeared first on GMAT.
This Blog post was imported into the forum automatically. We hope you found it helpful. Please use the Kudos button if you did, or please PM/DM me if you found it disruptive and I will take care of it. -BB
Manhattan GMAT Online Marketing Associate
Joined: 14 Nov 2013
Posts: 272
Own Kudos [?]: 51 [0]
Given Kudos: 0
Send PM
GMAT Grammar Biweekly: Adverbial Modifiers [#permalink]
Expert Reply
FROM Manhattan GMAT Blog: GMAT Grammar Biweekly: Adverbial Modifiers



Did you know that you can attend the first session of any of our online or in-person GMAT courses absolutely free? We’re not kidding! Check out our upcoming courses here.



Have you been following our grammar tips series? We’ve already talked about opening modifiers and noun modifiers. We’re almost done with this much-feared topic. If you’re still having problems, it’s probably with adverbial modifiers.

These can be the most overwhelming, so let’s break them down now. Back to our favorite modifier-riddled sentence:

Barking ferociously, the dog, which was known to be vicious, ran down the street, chasing the boy who had been poking at it just moments before.

An adverbial modifier is something that describes almost anything in the world that is not a noun. There’s actually a one-word adverbial modifier in our ferocious dog sentence (or, put far more simply, an adverb). Go back and see if you can find it.It’s the second word in the sentence; ferociously. But “barking ferociously” is a noun modifier. “Ferociously” is an adverbial modifier. If you’re wearing a confused/exasperated/annoyed expression, you’re not alone. Don’t worry – the GMAT isn’t going to go so crazy as to have you dissect clauses word-by-word, but we can use this adverb to start explaining the role of an adverbial modifier.

Step back and try to focus on meaning. What, exactly, is “ferociously”? Your first instinct may be the dog, but there’s a problem with that instinct. Take a moment and create a sentence that describes a dog using that word. Here’s what I came up with:

The dog is ferociously.

Wait. Full stop. What I just wrote sounds really wrong. And it is really wrong! The sentence that makes sense is:

The dog is ferocious.

A dog can be ferocious, but cannot be ferociously. Go back to the original sentence and see if you can now find something that is described as “ferociously.”

It’s the way the dog is barking! The dog is barking ferociously. Adverbial modifiers describe verbs, adjective, clauses, and anything else that is not a noun.

The GMAT will often test adverbial modifiers using present participles. Translated for the non-grammarian: words ending in –ing. If you see “, -ing” think about what the –ing is describing. It is often the entire clause before the comma. Practice IDing what is being described by the adverbial modifier in each of these sentences:

  • The girl sat near the river, cooling her feet in the water.
  • Companies are more likely to report positive outcomes than negative ones, causing investors to make suspect decisions, though the information is available to those who look hard enough.
  • After taking up drumming, John seemed to constantly get complaints from the neighbors, forcing him to soundproof his garage.
What’s being described by each –ing modifier is more complex than we’ve seen with noun modifiers. Here’s a basic breakdown of the meaning:

  • “Cooling her feet” is modifying how the girl sits. Think of it as a result. She sits. Therefore, her feet are cooled. The cooling is describing what is happening because she’s sitting, not just the girl.
  • What is “causing investors to make suspect decisions”? That companies skew their reporting. Again, the adverbial modifier is describing what happens because of the action, not just describing the company or the reports.
  • The third is the same as the first two. What forced the soundproofing? Getting complaints. A large idea is being described, so we’re dealing with adverbial modifiers.
Seem simple enough? Look at the meaning to figure out what is being modified. If it’s an idea that’s larger than a noun, insert an adverbial modifier. Before we wrap up, let’s take a look at how the GMAT might throw in some flawed sentences. See if you can find and fix the errors:

[*]Everyone was enjoying the party, except Susan, calling her best friend and complaining about the evening.[*]Mike developed a game called Gongu, which pushed him to the limits of his developing ability.[*]Jordan and Kyle, racing against each other, were both in fourth grade.[/list]
All of these are GMAT-wrong, though some may have sounded right, and some may even be sentences you would happily say in actual speech. Let’s break this down GMAT style.

[*]“Calling her best friend…” describes Susan. This is a problem because Susan is clearly a noun (don’t you think “noun” every time you see Susan?), but the modifier is adverbial. Let’s swap it out for a noun modifier.[/list]
Everyone was enjoying the party, except Susan, who was calling her best friend and complaining about the evening.

[*]What pushed Mike to his limits? There’s two possible answers. Either the game itself, Gongu, or the act of developing the game. Fortunately, the sentence gives you a hint about which is correct. It’s his developing ability, so it makes sense that developing the game was challenging, not necessarily playing it. That’s not a simple noun, it’s a verb (with some extra specifics tacked on). “, which” is a noun modifier, so, again, we need to make a swap.[/list]
Mike developed a game called Gongu, pushing himself to the limits of his developing ability.

[*]This can be a toughie, even though it’s the shortest sentence. Racing is meant to describe the boys, but can’t because it’s adverbial. The easy fix is to make it a noun modifier:[/list]
Jordan and Kyle, who were racing against each other, were both in fourth grade.

but let’s not take the easy way out. How can you change the sentence so that an adverbial modifier works? You’d have to make “racing…” modify the entire clause, not just the boys.

Jordan and Kyle, racing against each other, showed their competitive spirits.

Now the adverbial modifier works because racing illustrates how the boys are showing their competitive sides, not just the boys themselves.

Adverbial modifiers can be tricky, but focus on their intended meaning, then compare that to actual meaning. If you want a more thorough discussion of modifiers, visit Chapter 6 of our Sentence Correction Strategy Guide

Of course, the most in-depth way to learn the ins-and-outs of the GMAT is to take a complete course with one of our master instructors. You can try out any first session for free! No strings attached. We promise.


Emily Madan is a Manhattan Prep instructor based in Philadelphia. Having scored in the 99th percentile of the GMAT (770) and LSAT (177), Emily is committed to helping others achieve their full potential. In the classroom, she loves bringing concepts to life and her greatest thrill is that moment when a complex topic suddenly becomes clear to her students. Check out Emily’s upcoming GMAT courses here. Your first class is always free!

The post GMAT Grammar Biweekly: Adverbial Modifiers appeared first on GMAT.
This Blog post was imported into the forum automatically. We hope you found it helpful. Please use the Kudos button if you did, or please PM/DM me if you found it disruptive and I will take care of it. -BB
Manhattan GMAT Online Marketing Associate
Joined: 14 Nov 2013
Posts: 272
Own Kudos [?]: 51 [0]
Given Kudos: 0
Send PM
Here’s What to Do When You Can’t Find the “Split” on GMAT Sentence Cor [#permalink]
Expert Reply
FROM Manhattan GMAT Blog: Here’s What to Do When You Can’t Find the “Split” on GMAT Sentence Correction

Did you know that you can attend the first session of any of our online or in-person GMAT courses absolutely free? We’re not kidding! Check out our upcoming courses here.

In GMAT Sentence Correction, a “split” is a clear difference among the answer choices that allows you to identify and eliminate several incorrect answers. You can’t always find a perfect, straightforward answer choice split to work with in every Sentence Correction problem. Sometimes, most or all of the sentence is underlined, and the answer choices seem completely different from each other. When this happens, don’t fall back on bad habits. Even if you can’t find a great split, you can take a smart, fast approach to the problem. Let’s work through that approach using the following problem, from the GMAC’s GMAT Prep software.

In Holland, a larger percentage of the gross national product is spent on defense of their coasts from rising seas than is spent on military defense in the United States.

(A) In Holland, a larger percentage of the gross national product is spent on defense of their coasts from rising seas than is spent on military defense in the United States.

(B) In Holland they spend a larger percentage of their gross national product on defending their coasts from rising seas than the United States does on military defense.

(C) A larger percentage of Holland’s gross national product is spent on defending their coasts from rising seas than the United States spends on military defense.

(D) Holland spends a larger percentage of its gross national product defending its coasts from rising seas than the military defense spending of the United States.

(E) Holland spends a larger percentage of its gross national product on defending its coasts from rising seas than the United States does on military defense.

Read the original sentence carefully. The GMAT tests the same grammatical rules over and over, and the test writers write Sentence Correction problems that showcase these rules. Regardless of whether it’s grammatically correct, the original sentence will contain modifiers, pronouns, lists, or other GMAT-like features.

Choose one of these interesting features in the original sentence. It doesn’t have to be an error, although it’s fine if it is. Pick something that you’re comfortable working with. The best candidate in this problem is the pronoun their in the original sentence. The rules of pronouns are easy to remember, and it’s easy to scan the sentence for pronouns quickly.

In Holland, a larger percentage of the gross national product is spent on defense of their coasts from rising seas than is spent on military defense in the United States.

In this case, their is an error. The plural pronoun their is only grammatical when it refers to a specific plural noun in the sentence. In this case, it refers to ‘people from Holland’ or ‘government officials from Holland’, neither of which appear in the sentence.

Your next task is to examine how each answer choice handles this issue. It might handle it the same way as the original sentence. It might do something different. Or, it might even avoid the issue entirely by restructuring the sentence.

(B) In Holland they spend a larger percentage of their gross national product on defending their coasts from rising seas than the United States does on military defense.

(C) A larger percentage of Holland’s gross national product is spent on defending their coasts from rising seas than the United States spends on military defense.

On investigation, (B) and (C) use the exact same pronoun, even though it’s in a different location. Eliminate them with confidence! Once you find an error in an answer choice, no matter how small, never consider that option again.

(D) Holland spends a larger percentage of its gross national product defending its coasts from rising seas than the military defense spending of the United States.

(E) Holland spends a larger percentage of its gross national product on defending its coasts from rising seas than the United States does on military defense.

(D) and (E), however, use the correct pronoun its.

If you still had three or more answer choices remaining, you’d pick a second feature and check it against all of the remaining options. Here, though, only two answer choices remain. At this point, you should compare the two answer choices directly against each other. Identify exactly what they do differently, and then decide which one is correct.

The only real difference between these two sentences is how they structure the comparison. Compare them directly against each other by simplifying them.

(D) Holland spends a larger percentage… than the spending of the US.

(E) Holland spends a larger percentage… than the US does.

Only one of these will be correct. In this case, (E) is grammatical. It compares a sentence about Holland (‘Holland spends…’) to a sentence about the US (‘the US does’). (D) is ungrammatical, because it compares the same sentence about Holland to a noun describing an amount of spending (‘the spending of the US’). So, the right answer is (E).

Let’s review the key principles of handling long, complicated SC problems without obvious splits:

  • You don’t need to see a split in every answer choice immediately. It’s okay to start with something interesting that you notice in a single answer choice, and work from there.
  • Pick a ‘feature’ of the first answer choice, such as a pronoun, a modifier, or a tensed verb, and check how each answer choice handles it. Eliminate any answer that handles it incorrectly.
  • Choose easy features first. Only work with difficult grammar rules if you have to.
  • Eliminate wrong answers completely. If a sentence has a single incorrect element, you can ignore it from then on.
  • Even the toughest SC problems have a process. Don’t let the test intimidate you into working carelessly. Instead, when you see a problem like this, remind yourself that there is a fast way to solve it. You just need to find it! Take a deep breath, and start looking for the best place to start.
Of course, there are many more nuances to GMAT Sentence Correction than can be covered in a single blog post. For further reading, check out our GMAT Sentence Correction Strategy Guide

Want full access to Chelsey’s sage GMAT wisdom? Try the first class of one of her upcoming GMAT courses for absolutely free, no strings attached. 

Chelsey Cooley
is a Manhattan Prep instructor based in Seattle, Washington.
 Chelsey always followed her heart when it came to her education. Luckily, her heart led her straight to the perfect background for GMAT and GRE teaching: she has undergraduate degrees in mathematics and history, a master’s degree in linguistics, a 790 on the GMAT, and a perfect 170/170 on the GRE. Check out Chelsey’s upcoming GMAT prep offerings here.

The post Here’s What to Do When You Can’t Find the “Split” on GMAT Sentence Correction appeared first on GMAT.
This Blog post was imported into the forum automatically. We hope you found it helpful. Please use the Kudos button if you did, or please PM/DM me if you found it disruptive and I will take care of it. -BB
GMAT Club Bot
Here’s What to Do When You Can’t Find the “Split” on GMAT Sentence Cor [#permalink]
   1  ...  7   8   9   10   11  ...  21